Is $1 + frac{1}{2} + frac{1}{3} + dots + frac{1}{x} < log_2 x$











up vote
2
down vote

favorite
1












If $x ge 5$, is $1 + frac{1}{2} + frac{1}{3} + dots + frac{1}{x} < log_2 x$



I believe the answer is yes.



Here is my thinking:



(1) $log_2{5} > 2.32 > 2.284 > 1 + frac{1}{2} + frac{1}{3} + frac{1}{4} + frac{1}{5}$



(2) Assume up to $x$ that $log_2 x > sumlimits_{i le x}frac{1}{i}$



(3) $(x+1)^{x+1} > {{x+1}choose{x+1}}x^{x+1} + {{x+1}choose{x}}x^x = x^{x+1} + (x+1)x^x = 2x^{x+1} + x^x > 2(x^{x+1})$



(4) $(x+1)log_2left(frac{x+1}{x}right) > 1$



(5) $log_2(x+1) - log_2(x) > frac{1}{x+1}$



(6) $log_2(x+1) - log_2(x) + log_2(x) = log_2(x+1) > sumlimits_{i le x+1}frac{1}{i}$










share|cite|improve this question






















  • See this Wikipedia article, then consider $ln x= ln 2cdot log_2 x$
    – Arthur
    Nov 23 at 7:59












  • Thanks, Arthur but I'm not clear if you are saying it is true or false. $ln 2 < 0.694$ so that $ln 2 < log_2 x$. The article shows that $ln x < 1 + 1/2 + dots + 1/x$. I am not clear what that says about my question.
    – Larry Freeman
    Nov 23 at 8:07










  • It also shows that $1+ln xgeq1+1/2+cdots$.
    – Arthur
    Nov 23 at 8:10












  • ok. Now I get it. You are saying it is true because for $x > 11, log_2 x > (ln x)frac{10}{7} > ln x + 1$ Thanks.
    – Larry Freeman
    Nov 23 at 8:17










  • And then you can check by hand which cases $xleq 11$ it is true for as well. Exactly.
    – Arthur
    Nov 23 at 8:23

















up vote
2
down vote

favorite
1












If $x ge 5$, is $1 + frac{1}{2} + frac{1}{3} + dots + frac{1}{x} < log_2 x$



I believe the answer is yes.



Here is my thinking:



(1) $log_2{5} > 2.32 > 2.284 > 1 + frac{1}{2} + frac{1}{3} + frac{1}{4} + frac{1}{5}$



(2) Assume up to $x$ that $log_2 x > sumlimits_{i le x}frac{1}{i}$



(3) $(x+1)^{x+1} > {{x+1}choose{x+1}}x^{x+1} + {{x+1}choose{x}}x^x = x^{x+1} + (x+1)x^x = 2x^{x+1} + x^x > 2(x^{x+1})$



(4) $(x+1)log_2left(frac{x+1}{x}right) > 1$



(5) $log_2(x+1) - log_2(x) > frac{1}{x+1}$



(6) $log_2(x+1) - log_2(x) + log_2(x) = log_2(x+1) > sumlimits_{i le x+1}frac{1}{i}$










share|cite|improve this question






















  • See this Wikipedia article, then consider $ln x= ln 2cdot log_2 x$
    – Arthur
    Nov 23 at 7:59












  • Thanks, Arthur but I'm not clear if you are saying it is true or false. $ln 2 < 0.694$ so that $ln 2 < log_2 x$. The article shows that $ln x < 1 + 1/2 + dots + 1/x$. I am not clear what that says about my question.
    – Larry Freeman
    Nov 23 at 8:07










  • It also shows that $1+ln xgeq1+1/2+cdots$.
    – Arthur
    Nov 23 at 8:10












  • ok. Now I get it. You are saying it is true because for $x > 11, log_2 x > (ln x)frac{10}{7} > ln x + 1$ Thanks.
    – Larry Freeman
    Nov 23 at 8:17










  • And then you can check by hand which cases $xleq 11$ it is true for as well. Exactly.
    – Arthur
    Nov 23 at 8:23















up vote
2
down vote

favorite
1









up vote
2
down vote

favorite
1






1





If $x ge 5$, is $1 + frac{1}{2} + frac{1}{3} + dots + frac{1}{x} < log_2 x$



I believe the answer is yes.



Here is my thinking:



(1) $log_2{5} > 2.32 > 2.284 > 1 + frac{1}{2} + frac{1}{3} + frac{1}{4} + frac{1}{5}$



(2) Assume up to $x$ that $log_2 x > sumlimits_{i le x}frac{1}{i}$



(3) $(x+1)^{x+1} > {{x+1}choose{x+1}}x^{x+1} + {{x+1}choose{x}}x^x = x^{x+1} + (x+1)x^x = 2x^{x+1} + x^x > 2(x^{x+1})$



(4) $(x+1)log_2left(frac{x+1}{x}right) > 1$



(5) $log_2(x+1) - log_2(x) > frac{1}{x+1}$



(6) $log_2(x+1) - log_2(x) + log_2(x) = log_2(x+1) > sumlimits_{i le x+1}frac{1}{i}$










share|cite|improve this question













If $x ge 5$, is $1 + frac{1}{2} + frac{1}{3} + dots + frac{1}{x} < log_2 x$



I believe the answer is yes.



Here is my thinking:



(1) $log_2{5} > 2.32 > 2.284 > 1 + frac{1}{2} + frac{1}{3} + frac{1}{4} + frac{1}{5}$



(2) Assume up to $x$ that $log_2 x > sumlimits_{i le x}frac{1}{i}$



(3) $(x+1)^{x+1} > {{x+1}choose{x+1}}x^{x+1} + {{x+1}choose{x}}x^x = x^{x+1} + (x+1)x^x = 2x^{x+1} + x^x > 2(x^{x+1})$



(4) $(x+1)log_2left(frac{x+1}{x}right) > 1$



(5) $log_2(x+1) - log_2(x) > frac{1}{x+1}$



(6) $log_2(x+1) - log_2(x) + log_2(x) = log_2(x+1) > sumlimits_{i le x+1}frac{1}{i}$







proof-verification inequality logarithms harmonic-numbers






share|cite|improve this question













share|cite|improve this question











share|cite|improve this question




share|cite|improve this question










asked Nov 23 at 7:55









Larry Freeman

3,23421239




3,23421239












  • See this Wikipedia article, then consider $ln x= ln 2cdot log_2 x$
    – Arthur
    Nov 23 at 7:59












  • Thanks, Arthur but I'm not clear if you are saying it is true or false. $ln 2 < 0.694$ so that $ln 2 < log_2 x$. The article shows that $ln x < 1 + 1/2 + dots + 1/x$. I am not clear what that says about my question.
    – Larry Freeman
    Nov 23 at 8:07










  • It also shows that $1+ln xgeq1+1/2+cdots$.
    – Arthur
    Nov 23 at 8:10












  • ok. Now I get it. You are saying it is true because for $x > 11, log_2 x > (ln x)frac{10}{7} > ln x + 1$ Thanks.
    – Larry Freeman
    Nov 23 at 8:17










  • And then you can check by hand which cases $xleq 11$ it is true for as well. Exactly.
    – Arthur
    Nov 23 at 8:23




















  • See this Wikipedia article, then consider $ln x= ln 2cdot log_2 x$
    – Arthur
    Nov 23 at 7:59












  • Thanks, Arthur but I'm not clear if you are saying it is true or false. $ln 2 < 0.694$ so that $ln 2 < log_2 x$. The article shows that $ln x < 1 + 1/2 + dots + 1/x$. I am not clear what that says about my question.
    – Larry Freeman
    Nov 23 at 8:07










  • It also shows that $1+ln xgeq1+1/2+cdots$.
    – Arthur
    Nov 23 at 8:10












  • ok. Now I get it. You are saying it is true because for $x > 11, log_2 x > (ln x)frac{10}{7} > ln x + 1$ Thanks.
    – Larry Freeman
    Nov 23 at 8:17










  • And then you can check by hand which cases $xleq 11$ it is true for as well. Exactly.
    – Arthur
    Nov 23 at 8:23


















See this Wikipedia article, then consider $ln x= ln 2cdot log_2 x$
– Arthur
Nov 23 at 7:59






See this Wikipedia article, then consider $ln x= ln 2cdot log_2 x$
– Arthur
Nov 23 at 7:59














Thanks, Arthur but I'm not clear if you are saying it is true or false. $ln 2 < 0.694$ so that $ln 2 < log_2 x$. The article shows that $ln x < 1 + 1/2 + dots + 1/x$. I am not clear what that says about my question.
– Larry Freeman
Nov 23 at 8:07




Thanks, Arthur but I'm not clear if you are saying it is true or false. $ln 2 < 0.694$ so that $ln 2 < log_2 x$. The article shows that $ln x < 1 + 1/2 + dots + 1/x$. I am not clear what that says about my question.
– Larry Freeman
Nov 23 at 8:07












It also shows that $1+ln xgeq1+1/2+cdots$.
– Arthur
Nov 23 at 8:10






It also shows that $1+ln xgeq1+1/2+cdots$.
– Arthur
Nov 23 at 8:10














ok. Now I get it. You are saying it is true because for $x > 11, log_2 x > (ln x)frac{10}{7} > ln x + 1$ Thanks.
– Larry Freeman
Nov 23 at 8:17




ok. Now I get it. You are saying it is true because for $x > 11, log_2 x > (ln x)frac{10}{7} > ln x + 1$ Thanks.
– Larry Freeman
Nov 23 at 8:17












And then you can check by hand which cases $xleq 11$ it is true for as well. Exactly.
– Arthur
Nov 23 at 8:23






And then you can check by hand which cases $xleq 11$ it is true for as well. Exactly.
– Arthur
Nov 23 at 8:23












3 Answers
3






active

oldest

votes

















up vote
2
down vote



accepted










We have that



$$1 + frac{1}{2} + frac{1}{3} + dots + frac{1}{x} < log_2 x iff 1 + frac{1}{2} + frac{1}{3} + dots + frac{1}{x} < frac{ln x}{ln 2}$$



$$ln 2left(1 + frac{1}{2} + frac{1}{3} + dots + frac{1}{x} right)< ln x$$



then recall that by harmonic series



$$sum_{k=1}^x frac1k simln x+gamma+frac1{2x}$$



then



$$ln 2left(1 + frac{1}{2} + frac{1}{3} + dots + frac{1}{x} right) sim ln 2ln x+gammaln 2+frac{ln 2}{2x}stackrel{?}<ln x$$



$$(1-ln 2)ln x> gammaln 2+frac{ln 2}{2x}$$



which is true for $x$ sufficiently large.






share|cite|improve this answer























  • After "harmonic series" your summation has an error (the summand should be $1/k$)
    – YiFan
    Nov 23 at 9:29










  • @YiFan Opssss...yes of course I fix the typo. Thanks!
    – gimusi
    Nov 23 at 9:45


















up vote
1
down vote













We can use induction as well. If we assume that it's true for $n$:
$$1+frac{1}{2}+...+frac{1}{n}+frac{1}{n+1}<log_2(n)+frac{1}{n+1}$$
So we need to prove that
$$log_2(n)+frac{1}{n+1} < log_2(n+1)$$
$$frac{1}{n+1}< log_2left(1+frac{1}{n}right)$$
$$2^{frac{1}{n+1}}<1+frac{1}{n}$$
$$2<left(1+frac{1}{n}right)^{n+1}$$
$$2<left(1+frac{1}{n}right) left(1+frac{1}{n}right)^n$$
And it's true, because
$$1+frac{1}{n}>1$$
And by Bernoulli's inequality:
$$left(1+frac{1}{n}right)^n geq 1+n frac{1}{n}=2$$
So we just need to find a good starting $n$. Let's check it for $n=6$:
$$1+frac{1}{2}+frac{1}{3}+frac{1}{4}+frac{1}{5}+frac{1}{6}<log_2(6)$$
$$frac{9}{20}<log_2({6/4})$$
$$2^{frac{9}{20}}<frac{6}{4}$$
And by Bernoulli's inequality,
$$2^{frac{9}{20}}<1+frac{9}{20}<1+frac{10}{20}=frac{6}{4}$$
So it's true $forall ngeq 6 land n in mathbb{N}$ (it's true for $n=5$ as well, but Bernoulli is not helping in that case).






share|cite|improve this answer






























    up vote
    1
    down vote













    Yes because $H_n-log n$ is bounded by $1$ and $log_2n$ is a multiple of $log n$.






    share|cite|improve this answer





















      Your Answer





      StackExchange.ifUsing("editor", function () {
      return StackExchange.using("mathjaxEditing", function () {
      StackExchange.MarkdownEditor.creationCallbacks.add(function (editor, postfix) {
      StackExchange.mathjaxEditing.prepareWmdForMathJax(editor, postfix, [["$", "$"], ["\\(","\\)"]]);
      });
      });
      }, "mathjax-editing");

      StackExchange.ready(function() {
      var channelOptions = {
      tags: "".split(" "),
      id: "69"
      };
      initTagRenderer("".split(" "), "".split(" "), channelOptions);

      StackExchange.using("externalEditor", function() {
      // Have to fire editor after snippets, if snippets enabled
      if (StackExchange.settings.snippets.snippetsEnabled) {
      StackExchange.using("snippets", function() {
      createEditor();
      });
      }
      else {
      createEditor();
      }
      });

      function createEditor() {
      StackExchange.prepareEditor({
      heartbeatType: 'answer',
      convertImagesToLinks: true,
      noModals: true,
      showLowRepImageUploadWarning: true,
      reputationToPostImages: 10,
      bindNavPrevention: true,
      postfix: "",
      imageUploader: {
      brandingHtml: "Powered by u003ca class="icon-imgur-white" href="https://imgur.com/"u003eu003c/au003e",
      contentPolicyHtml: "User contributions licensed under u003ca href="https://creativecommons.org/licenses/by-sa/3.0/"u003ecc by-sa 3.0 with attribution requiredu003c/au003e u003ca href="https://stackoverflow.com/legal/content-policy"u003e(content policy)u003c/au003e",
      allowUrls: true
      },
      noCode: true, onDemand: true,
      discardSelector: ".discard-answer"
      ,immediatelyShowMarkdownHelp:true
      });


      }
      });














      draft saved

      draft discarded


















      StackExchange.ready(
      function () {
      StackExchange.openid.initPostLogin('.new-post-login', 'https%3a%2f%2fmath.stackexchange.com%2fquestions%2f3010094%2fis-1-frac12-frac13-dots-frac1x-log-2-x%23new-answer', 'question_page');
      }
      );

      Post as a guest















      Required, but never shown

























      3 Answers
      3






      active

      oldest

      votes








      3 Answers
      3






      active

      oldest

      votes









      active

      oldest

      votes






      active

      oldest

      votes








      up vote
      2
      down vote



      accepted










      We have that



      $$1 + frac{1}{2} + frac{1}{3} + dots + frac{1}{x} < log_2 x iff 1 + frac{1}{2} + frac{1}{3} + dots + frac{1}{x} < frac{ln x}{ln 2}$$



      $$ln 2left(1 + frac{1}{2} + frac{1}{3} + dots + frac{1}{x} right)< ln x$$



      then recall that by harmonic series



      $$sum_{k=1}^x frac1k simln x+gamma+frac1{2x}$$



      then



      $$ln 2left(1 + frac{1}{2} + frac{1}{3} + dots + frac{1}{x} right) sim ln 2ln x+gammaln 2+frac{ln 2}{2x}stackrel{?}<ln x$$



      $$(1-ln 2)ln x> gammaln 2+frac{ln 2}{2x}$$



      which is true for $x$ sufficiently large.






      share|cite|improve this answer























      • After "harmonic series" your summation has an error (the summand should be $1/k$)
        – YiFan
        Nov 23 at 9:29










      • @YiFan Opssss...yes of course I fix the typo. Thanks!
        – gimusi
        Nov 23 at 9:45















      up vote
      2
      down vote



      accepted










      We have that



      $$1 + frac{1}{2} + frac{1}{3} + dots + frac{1}{x} < log_2 x iff 1 + frac{1}{2} + frac{1}{3} + dots + frac{1}{x} < frac{ln x}{ln 2}$$



      $$ln 2left(1 + frac{1}{2} + frac{1}{3} + dots + frac{1}{x} right)< ln x$$



      then recall that by harmonic series



      $$sum_{k=1}^x frac1k simln x+gamma+frac1{2x}$$



      then



      $$ln 2left(1 + frac{1}{2} + frac{1}{3} + dots + frac{1}{x} right) sim ln 2ln x+gammaln 2+frac{ln 2}{2x}stackrel{?}<ln x$$



      $$(1-ln 2)ln x> gammaln 2+frac{ln 2}{2x}$$



      which is true for $x$ sufficiently large.






      share|cite|improve this answer























      • After "harmonic series" your summation has an error (the summand should be $1/k$)
        – YiFan
        Nov 23 at 9:29










      • @YiFan Opssss...yes of course I fix the typo. Thanks!
        – gimusi
        Nov 23 at 9:45













      up vote
      2
      down vote



      accepted







      up vote
      2
      down vote



      accepted






      We have that



      $$1 + frac{1}{2} + frac{1}{3} + dots + frac{1}{x} < log_2 x iff 1 + frac{1}{2} + frac{1}{3} + dots + frac{1}{x} < frac{ln x}{ln 2}$$



      $$ln 2left(1 + frac{1}{2} + frac{1}{3} + dots + frac{1}{x} right)< ln x$$



      then recall that by harmonic series



      $$sum_{k=1}^x frac1k simln x+gamma+frac1{2x}$$



      then



      $$ln 2left(1 + frac{1}{2} + frac{1}{3} + dots + frac{1}{x} right) sim ln 2ln x+gammaln 2+frac{ln 2}{2x}stackrel{?}<ln x$$



      $$(1-ln 2)ln x> gammaln 2+frac{ln 2}{2x}$$



      which is true for $x$ sufficiently large.






      share|cite|improve this answer














      We have that



      $$1 + frac{1}{2} + frac{1}{3} + dots + frac{1}{x} < log_2 x iff 1 + frac{1}{2} + frac{1}{3} + dots + frac{1}{x} < frac{ln x}{ln 2}$$



      $$ln 2left(1 + frac{1}{2} + frac{1}{3} + dots + frac{1}{x} right)< ln x$$



      then recall that by harmonic series



      $$sum_{k=1}^x frac1k simln x+gamma+frac1{2x}$$



      then



      $$ln 2left(1 + frac{1}{2} + frac{1}{3} + dots + frac{1}{x} right) sim ln 2ln x+gammaln 2+frac{ln 2}{2x}stackrel{?}<ln x$$



      $$(1-ln 2)ln x> gammaln 2+frac{ln 2}{2x}$$



      which is true for $x$ sufficiently large.







      share|cite|improve this answer














      share|cite|improve this answer



      share|cite|improve this answer








      edited Nov 23 at 9:46

























      answered Nov 23 at 8:42









      gimusi

      92.3k84495




      92.3k84495












      • After "harmonic series" your summation has an error (the summand should be $1/k$)
        – YiFan
        Nov 23 at 9:29










      • @YiFan Opssss...yes of course I fix the typo. Thanks!
        – gimusi
        Nov 23 at 9:45


















      • After "harmonic series" your summation has an error (the summand should be $1/k$)
        – YiFan
        Nov 23 at 9:29










      • @YiFan Opssss...yes of course I fix the typo. Thanks!
        – gimusi
        Nov 23 at 9:45
















      After "harmonic series" your summation has an error (the summand should be $1/k$)
      – YiFan
      Nov 23 at 9:29




      After "harmonic series" your summation has an error (the summand should be $1/k$)
      – YiFan
      Nov 23 at 9:29












      @YiFan Opssss...yes of course I fix the typo. Thanks!
      – gimusi
      Nov 23 at 9:45




      @YiFan Opssss...yes of course I fix the typo. Thanks!
      – gimusi
      Nov 23 at 9:45










      up vote
      1
      down vote













      We can use induction as well. If we assume that it's true for $n$:
      $$1+frac{1}{2}+...+frac{1}{n}+frac{1}{n+1}<log_2(n)+frac{1}{n+1}$$
      So we need to prove that
      $$log_2(n)+frac{1}{n+1} < log_2(n+1)$$
      $$frac{1}{n+1}< log_2left(1+frac{1}{n}right)$$
      $$2^{frac{1}{n+1}}<1+frac{1}{n}$$
      $$2<left(1+frac{1}{n}right)^{n+1}$$
      $$2<left(1+frac{1}{n}right) left(1+frac{1}{n}right)^n$$
      And it's true, because
      $$1+frac{1}{n}>1$$
      And by Bernoulli's inequality:
      $$left(1+frac{1}{n}right)^n geq 1+n frac{1}{n}=2$$
      So we just need to find a good starting $n$. Let's check it for $n=6$:
      $$1+frac{1}{2}+frac{1}{3}+frac{1}{4}+frac{1}{5}+frac{1}{6}<log_2(6)$$
      $$frac{9}{20}<log_2({6/4})$$
      $$2^{frac{9}{20}}<frac{6}{4}$$
      And by Bernoulli's inequality,
      $$2^{frac{9}{20}}<1+frac{9}{20}<1+frac{10}{20}=frac{6}{4}$$
      So it's true $forall ngeq 6 land n in mathbb{N}$ (it's true for $n=5$ as well, but Bernoulli is not helping in that case).






      share|cite|improve this answer



























        up vote
        1
        down vote













        We can use induction as well. If we assume that it's true for $n$:
        $$1+frac{1}{2}+...+frac{1}{n}+frac{1}{n+1}<log_2(n)+frac{1}{n+1}$$
        So we need to prove that
        $$log_2(n)+frac{1}{n+1} < log_2(n+1)$$
        $$frac{1}{n+1}< log_2left(1+frac{1}{n}right)$$
        $$2^{frac{1}{n+1}}<1+frac{1}{n}$$
        $$2<left(1+frac{1}{n}right)^{n+1}$$
        $$2<left(1+frac{1}{n}right) left(1+frac{1}{n}right)^n$$
        And it's true, because
        $$1+frac{1}{n}>1$$
        And by Bernoulli's inequality:
        $$left(1+frac{1}{n}right)^n geq 1+n frac{1}{n}=2$$
        So we just need to find a good starting $n$. Let's check it for $n=6$:
        $$1+frac{1}{2}+frac{1}{3}+frac{1}{4}+frac{1}{5}+frac{1}{6}<log_2(6)$$
        $$frac{9}{20}<log_2({6/4})$$
        $$2^{frac{9}{20}}<frac{6}{4}$$
        And by Bernoulli's inequality,
        $$2^{frac{9}{20}}<1+frac{9}{20}<1+frac{10}{20}=frac{6}{4}$$
        So it's true $forall ngeq 6 land n in mathbb{N}$ (it's true for $n=5$ as well, but Bernoulli is not helping in that case).






        share|cite|improve this answer

























          up vote
          1
          down vote










          up vote
          1
          down vote









          We can use induction as well. If we assume that it's true for $n$:
          $$1+frac{1}{2}+...+frac{1}{n}+frac{1}{n+1}<log_2(n)+frac{1}{n+1}$$
          So we need to prove that
          $$log_2(n)+frac{1}{n+1} < log_2(n+1)$$
          $$frac{1}{n+1}< log_2left(1+frac{1}{n}right)$$
          $$2^{frac{1}{n+1}}<1+frac{1}{n}$$
          $$2<left(1+frac{1}{n}right)^{n+1}$$
          $$2<left(1+frac{1}{n}right) left(1+frac{1}{n}right)^n$$
          And it's true, because
          $$1+frac{1}{n}>1$$
          And by Bernoulli's inequality:
          $$left(1+frac{1}{n}right)^n geq 1+n frac{1}{n}=2$$
          So we just need to find a good starting $n$. Let's check it for $n=6$:
          $$1+frac{1}{2}+frac{1}{3}+frac{1}{4}+frac{1}{5}+frac{1}{6}<log_2(6)$$
          $$frac{9}{20}<log_2({6/4})$$
          $$2^{frac{9}{20}}<frac{6}{4}$$
          And by Bernoulli's inequality,
          $$2^{frac{9}{20}}<1+frac{9}{20}<1+frac{10}{20}=frac{6}{4}$$
          So it's true $forall ngeq 6 land n in mathbb{N}$ (it's true for $n=5$ as well, but Bernoulli is not helping in that case).






          share|cite|improve this answer














          We can use induction as well. If we assume that it's true for $n$:
          $$1+frac{1}{2}+...+frac{1}{n}+frac{1}{n+1}<log_2(n)+frac{1}{n+1}$$
          So we need to prove that
          $$log_2(n)+frac{1}{n+1} < log_2(n+1)$$
          $$frac{1}{n+1}< log_2left(1+frac{1}{n}right)$$
          $$2^{frac{1}{n+1}}<1+frac{1}{n}$$
          $$2<left(1+frac{1}{n}right)^{n+1}$$
          $$2<left(1+frac{1}{n}right) left(1+frac{1}{n}right)^n$$
          And it's true, because
          $$1+frac{1}{n}>1$$
          And by Bernoulli's inequality:
          $$left(1+frac{1}{n}right)^n geq 1+n frac{1}{n}=2$$
          So we just need to find a good starting $n$. Let's check it for $n=6$:
          $$1+frac{1}{2}+frac{1}{3}+frac{1}{4}+frac{1}{5}+frac{1}{6}<log_2(6)$$
          $$frac{9}{20}<log_2({6/4})$$
          $$2^{frac{9}{20}}<frac{6}{4}$$
          And by Bernoulli's inequality,
          $$2^{frac{9}{20}}<1+frac{9}{20}<1+frac{10}{20}=frac{6}{4}$$
          So it's true $forall ngeq 6 land n in mathbb{N}$ (it's true for $n=5$ as well, but Bernoulli is not helping in that case).







          share|cite|improve this answer














          share|cite|improve this answer



          share|cite|improve this answer








          edited Nov 23 at 10:51

























          answered Nov 23 at 10:34









          Botond

          5,3712732




          5,3712732






















              up vote
              1
              down vote













              Yes because $H_n-log n$ is bounded by $1$ and $log_2n$ is a multiple of $log n$.






              share|cite|improve this answer

























                up vote
                1
                down vote













                Yes because $H_n-log n$ is bounded by $1$ and $log_2n$ is a multiple of $log n$.






                share|cite|improve this answer























                  up vote
                  1
                  down vote










                  up vote
                  1
                  down vote









                  Yes because $H_n-log n$ is bounded by $1$ and $log_2n$ is a multiple of $log n$.






                  share|cite|improve this answer












                  Yes because $H_n-log n$ is bounded by $1$ and $log_2n$ is a multiple of $log n$.







                  share|cite|improve this answer












                  share|cite|improve this answer



                  share|cite|improve this answer










                  answered Nov 23 at 10:58









                  Yves Daoust

                  123k668219




                  123k668219






























                      draft saved

                      draft discarded




















































                      Thanks for contributing an answer to Mathematics Stack Exchange!


                      • Please be sure to answer the question. Provide details and share your research!

                      But avoid



                      • Asking for help, clarification, or responding to other answers.

                      • Making statements based on opinion; back them up with references or personal experience.


                      Use MathJax to format equations. MathJax reference.


                      To learn more, see our tips on writing great answers.





                      Some of your past answers have not been well-received, and you're in danger of being blocked from answering.


                      Please pay close attention to the following guidance:


                      • Please be sure to answer the question. Provide details and share your research!

                      But avoid



                      • Asking for help, clarification, or responding to other answers.

                      • Making statements based on opinion; back them up with references or personal experience.


                      To learn more, see our tips on writing great answers.




                      draft saved


                      draft discarded














                      StackExchange.ready(
                      function () {
                      StackExchange.openid.initPostLogin('.new-post-login', 'https%3a%2f%2fmath.stackexchange.com%2fquestions%2f3010094%2fis-1-frac12-frac13-dots-frac1x-log-2-x%23new-answer', 'question_page');
                      }
                      );

                      Post as a guest















                      Required, but never shown





















































                      Required, but never shown














                      Required, but never shown












                      Required, but never shown







                      Required, but never shown

































                      Required, but never shown














                      Required, but never shown












                      Required, but never shown







                      Required, but never shown







                      Popular posts from this blog

                      Le Mesnil-Réaume

                      Ida-Boy-Ed-Garten

                      web3.py web3.isConnected() returns false always